PT75.S4.Q21 (G4)

meletzyoshermeletzyosher Free Trial Member
edited February 2018 in Logic Games 66 karma

I don't understand how E can be the correct answer. There actually appears to be no correct answer. The LSAT relies heavily on Formal Logic and applying a healthy dose of Logic to this answer dictates that every feature in slots 1-4 must contain either an "I" or an "M". Otherwise, you are a floater and can be anything. It should translate visually into this:
~5 ---> I or M
Alternatively, 1-4 ---> I or M.
In other words, slot 5 can also be I or M. The newspaper can have five M and one I without breaking any rules.
It appears a little bit like a double standard in this case as I can't see any way E conflicts with the rules using Formal Logic.

Comments

This discussion has been closed.